Prove that e^c = sinh(1) for some c in [-1,1]

  • Thread starter Thread starter ruby_duby
  • Start date Start date
Click For Summary
SUMMARY

The discussion centers on proving that \( e^c = \sinh(1) \) for some \( c \) in the interval \((-1, 1)\). The proof utilizes the Mean Value Theorem and the definition of the hyperbolic sine function, \( \sinh(x) = \frac{e^x - e^{-x}}{2} \). Participants clarify that while a specific value for \( c \) can be calculated as \( c = \ln(\sinh(1)) \), the primary objective is to demonstrate the existence of such a \( c \) rather than to compute it explicitly.

PREREQUISITES
  • Understanding of the Mean Value Theorem in calculus
  • Familiarity with the definition and properties of hyperbolic functions, specifically \( \sinh(x) \)
  • Knowledge of exponential functions and their derivatives
  • Basic skills in mathematical proof techniques
NEXT STEPS
  • Study the Mean Value Theorem and its applications in calculus
  • Explore the properties and graphs of hyperbolic functions, focusing on \( \sinh(x) \)
  • Learn how to derive and manipulate exponential functions and their derivatives
  • Practice constructing proofs in calculus, particularly those involving existence theorems
USEFUL FOR

Students studying calculus, particularly those interested in mathematical proofs and the properties of exponential and hyperbolic functions.

ruby_duby
Messages
46
Reaction score
0

Homework Statement



Assuming that (ex)’ = (ex) at all point x, prove that ec
= sinh(1) for some point C \in (-1,1). Give the full proof.

Homework Equations


The Attempt at a Solution



I honestly don't know how to attempt this. i can show that (ex)’ = (ex) by:

(ex) = 1 + x + x2/2! + x3/3! + x4! + x5/5! +...

(ex)’ = 1 + x + x2/2! + x3/3! + x4! + x5/5! +...

I just don't know how to tackle the sinh part of the equation.

I would really appreciate any help/ guidance
 
Physics news on Phys.org


Don't try series. Try Mean Value Theorem.

And don't forget the definition of sinh.
 


thanks so much, I've managed to work out the answer, however, does anyone know if i need to find a specific value for c?
 


Usually you don't, but if you do it's easy enough:

e^c=sinh(1), so c=ln(sinh(1))
 


ruby_duby said:
thanks so much, I've managed to work out the answer, however, does anyone know if i need to find a specific value for c?

You were only asked to show that it exists, not to actually give it (often in mathematics, it's easier to prove that you can construct or find it, than to actually do it).
 
Question: A clock's minute hand has length 4 and its hour hand has length 3. What is the distance between the tips at the moment when it is increasing most rapidly?(Putnam Exam Question) Answer: Making assumption that both the hands moves at constant angular velocities, the answer is ## \sqrt{7} .## But don't you think this assumption is somewhat doubtful and wrong?

Similar threads

  • · Replies 5 ·
Replies
5
Views
3K
  • · Replies 2 ·
Replies
2
Views
3K
  • · Replies 2 ·
Replies
2
Views
695
  • · Replies 1 ·
Replies
1
Views
863
Replies
2
Views
2K
  • · Replies 18 ·
Replies
18
Views
3K
  • · Replies 9 ·
Replies
9
Views
3K
  • · Replies 2 ·
Replies
2
Views
3K
  • · Replies 2 ·
Replies
2
Views
3K
Replies
10
Views
3K